0 Daumen
380 Aufrufe

Aufgabe:

Die Funktion \( f: \mathbb{R}^{3} \rightarrow \mathbb{R} \) sei definiert durch
\( f\left(x_{1}, x_{2}, x_{3}\right):=-x_{1}+x_{2}+2 x_{3} \)
Bestimmen Sie mithilfe des Verfahrens der Lagrange-Multiplikatoren die Extremstellen von \( f \) auf der Menge \( M:=\left\{x \in \mathbb{R}^{3}:\|x\|^{2}=1\right\} \). Sie können dabei davon ausgehen, dass sowohl eine Minimalstelle als auch eine Maximalstelle existieren.

Avatar von

1 Antwort

0 Daumen
 
Beste Antwort

Aloha :)

Willkommen in der Mathelounge... \o/

Es sollen alle Extremstellen der Funktion \(f\) unter einer konstanten Nebenbedingung \(g\) ermittelt werden:$$f(x;y;z)=-x+y+2z\quad;\quad g(x;y;z)=x^2+y^2+z^2\stackrel!=1$$Wir können laut Aufgabenstellung davon ausgehen, ein Minimum und ein Maximum zu finden, d.h. wir brauchen nicht mehr zu zeigen, dass es sich tatsächlich um Extremstellen handelt.

Um aber überhaupt erstmal die Kandidaten für Extremstellen zu finden, nutzen wir die Idee von Lagrange. Danach muss der Gradient der zu optimierenden Funktion eine Linearkombination der Gradienten aller Nebenbedingungen sein. Da wir hier nur eine Nebenbedingung haben, heißt das:$$\operatorname{grad}f(x;y;z)=\lambda\cdot\operatorname{grad}g(x;y;z)\quad\implies\quad\begin{pmatrix}-1\\1\\2\end{pmatrix}=\lambda\cdot\begin{pmatrix}2x\\2y\\2z\end{pmatrix}$$

Um den Lagrange-Multiplikator \(\lambda\) loszuwerden dividieren wir die Koordinatengleichungen:$$\frac{\lambda\cdot 2y}{\lambda\cdot 2x}=\frac{1}{-1}=-1\quad\implies\quad\frac{y}{x}=-1\quad\implies\quad y=-x$$$$\frac{\lambda\cdot2z}{\lambda\cdot2y}=\frac{2}{1}=2\quad\implies\quad\frac zy=2\quad\implies\quad z=2y=-2x$$

Wir setzen dies in die Nebenbedingung ein, um \(x\) zu ermitteln:$$1\stackrel!=x^2+(-x)^2+(-2x)^2=6x^2\quad\implies\quad x=\pm\frac{1}{\sqrt 6}$$

Wie erwartet haben wir die versprochenen zwei Kandidaten gefunden:$$K_1\left(-\frac{1}{\sqrt 6}\bigg|\frac{1}{\sqrt 6}\bigg|\frac{2}{\sqrt 6}\right)\quad;\quad K_2\left(\frac{1}{\sqrt 6}\bigg|-\frac{1}{\sqrt 6}\bigg|-\frac{2}{\sqrt 6}\right)$$

Durch EInsetzen finden wir die zugehörigen Funktionswerte \(\sqrt6\) und \(-\sqrt 6\). Daher ist \(K_1\) das Maximum und \(K_2\) das Minimum.

Avatar von 148 k 🚀

Ein anderes Problem?

Stell deine Frage

Willkommen bei der Mathelounge! Stell deine Frage einfach und kostenlos

x
Made by a lovely community